.widget.ContactForm { display: none; }

search

Friday 14 December 2012

Laporan Resmi Praktikum ALKALIMETRI KIMIA ANALISIS


PRAKTIKUM ALKALIMETRI

TUJUAN
Praktikan mampu menetapkan kadar CH3COOH (asam asetat) dan asam cuka (HCl) menggunakan prinsip reaksi asam-basa.

DASAR TEORI

Titrasi asam – basa adalah titrasi dimana reaksi antara titrat dan titranya merupakan reaksi asam – basa. Alkalimetri adalah penetapan kadar secara kuantitatif terhadap senyawa yang bersifat asam dengan menggunakan standar senyawa basa. Reaksi antara senyawa asam dan basa pada dasarnya adalah reaksi netralisasi, yaitu reaksi antara donor proton (asam) dengan resipien/aseptor proton (basa). Jika asam dan salah satu lemah maka garam akan terhidrolisa dan larutan sedikit asam/basa.
Asidi-alkalimetri merupakan salah satu metode kimia analisa kuantitatif yang didasarkan pada prinsip titrasi asam-basa. Asidi-alkalimetri berfungsi untuk menentukan kadar asam-basa dalam suatu larutan secara analisa volumetri. Titik akhir dari titrasi ini mudah dilihat dengan penambahan indikator yang sesuai. Percobaan ini dilakukan untuk menentukan kadar asam Cuka (CH3COOH) dengan titrasi Asidi-Alkalimetri. Sampai pH asam cuka berubah menjadi larutan basa, untuk ditentukan kadarnya.
Salah satu dari empat golongan utama dalam penggolongan analisis titrimetri adalah reaksi penetralan atau asidimetri dan alkalimetri. Asidi dan alkalimetri ini melibatkan titrasi basa yang terbentuk karena hidrolisis garam yang berasal dari asam lemah (basa bebas) dengan suatu asam standar (asidimetri), dan titrasi asam yang terbentuk dari hidrolisis garam yang berasal dari basa lemah (asam bebas) dengan suatu basa standar (alkalimetri). Bersenyawanya ion hidrogen dan ion hidroksida untuk membentuk air merupakan akibat reaksi-reaksi tersebut (Basset, J, 1994).
Larutan yang mengandung reagensia dengan bobot yang diketahui dalam suatu volume tertentu dalam suatu larutan disebut larutan standar. Sedangkan larutan standar primer adalah suatu larutan yang konsentrasinya dapat langsung ditentukan dari berat bahan sangat murni yang dilarutkan dan volume yang terjadi. Suatu zat standar primer harus memenuhi syarat seperti dibawah ini:
1. Zat harus mudah diperoleh, mudah dimurnikan, mudah dikeringkan (sebaiknya pada suhu 110-120oC).
2. Zat harus mempunyai ekuivalen yang tinggi, sehingga sesatan penimbangan dapat diabaikan.
3. Zat harus mudah larut pada kondisi-kondisi dalam mana ia digunakan.
4. Zat harus dapat diuji terhadap zat-zat pengotor dengan uji-uji kualitatif atau uji-uji lain yang kepekaannya diketahui (jumlah total zat-zat pengotor, umumnya tak boleh melebihi 0,01-0,02 %).
5. Reaksi dengan larutan standar itu harus stoikiometrik dan praktis sekejap. Sesatan titrasi harus dapat diabaikan, atau mudah ditetapkan dengan cermat dengan eksperimen.
6. Zat harus tak berubah dalam udara selama penimbangan; kondisi-kondisi ini mengisyaratkan bahwa zat tak boleh higroskopik, tak pula dioksidasi oleh udara, atau dipengaruhi oleh karbondioksida.Standar ini harus dijaga agar komposisinya tak berubah selama penyimpanan.
Natrium karbonat Na2CO3, natrium tetraborat Na2B4O7, kalium hydrogen iodat KH(IO3)2, asam klorida bertitik didih konstan merupakan zat-zat yang biasa digunakan sebagai standar primer. Sedangkan standar sekunder adalah suatu zat yang dapat digunakan untuk standarisasi yang kandungan zat aktifnya telah ditemukan dengan perbandingan terhadap suatu standar primer (Basset, J, 1994).
Proses penambahan larutan standar sampai reaksi tepat lengkap, disebut titrasi. Titik (saat) mana reaksi itu tepat lengkap, disebut titik ekuivalen (setara) atau titik akhir teoritis. Lengkapnya titrasi, lazimnya harus terdeteksi oleh suatu perubahan,yang tak dapat di salah lihat oleh mata, yang dihasilkan oleh larutan standar (biasanya ditambahkan dari dalam sebuah buret) itu sendiri, atau lebih lazim lagi, oleh penambahan suatu reagensia pembantu yang dikenal sebagai indikator (Basset, J, 1994).



Selama proses titrasi asam – basa, pH larutan terus menerus berubah dengan aturan yang khas. pH tersebut akan berubah secara drastis pada saat volume titran mendekati titik ekivalen.
Karakteristik dari kurva ini sangat penting, karena menentukan pemilihan indicator yang sesuai (paling mendekati titik ekivalen) untuk meminimalkan kesalahan titrasi. Indicator adalah zat yang berubah warnanya atau membentuk fluorescent pada suatu trayek pH tertentu. Perubahan ini terjadi karena karena adanya perubahan struktrur dari indicator tersebut.
Gambar diatas adalah contoh titrasi alkalimetri, terlihat bahwa pH naik perlahan terhadap penambahan NaOH. Pada saat mendekati titik ekivalen, pH menaik secara drastis. Berdasarkan hal tersebut, maka indikator yang sesuai adalah phenol phtalein yang bekerja pada trayek pH 8,3 -10. Phenol phtalein merupakan bentuk asam lemah yang lain. Asam lemah tidak berwarna dan ion-nya berwarna merah muda terang. Penambahan ion hidrogen berlebih menggeser posisi kesetimbangan ke arah kiri, dan mengubah indikator menjadi tak berwarna. Penambahan ion hidroksida menghilangkan ion hidrogen dari kesetimbangan yang mengarah ke kanan untuk menggantikannya - mengubah indikator menjadi ungu.
Selain dengan menggunakan indikator, titik ekivalen dapat dicari dengan bantuan pH meter. Kurva titrasi diperoleh dengan memplotkan data jumlah titran yang ditambahkan versus pH larutan. Titik ekivalen jelas terlihat dengan menggunakan perhitungan turunan kedua, dimana titik ekivalen merupakan perpotongan antara garis mendatar (volume titran).


data:image/png;base64,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
PROSEDUR KERJA
Alat dan Bahan yang Dipergunakan

Alat-alat yang digunakan adalah:

Neraca analitik
Gelas arloji 1 buah
Pipet gondok 10 ml 1 buah
Buret 25 ml 1 buah
Statif dan klem 1 buah
Corong gelas 2 buah (besar dan kecil)
Labu ukur 3 buah (50 ml, 100 ml, 250 ml)
Propipet 1 buah
Beker glass 200ml 1 buah
Pengaduk kaca 1 buah
Pipet tetes 1 buah
Botol semprot 1 buah
Erlenmeyer 250 ml 2 buah

Bahan-bahan yang digunakan adalah:

Aquades secukupnya
Sampel CH3COOH (asam asetat)
Sampel H2C2O4.2H2O (asam oksalat)
Sampel asam cuka
Sampel NaOH (natrium hidroksida)
Indicator PP (phenol phtalein)
Prosedur kerja
Standarisasi larutan NaOH
Semua alat yang akan di gunakan harus dibersihkan terlebih dahulu
Membuat larutan NaOH 0,1 N yang mana di fungsikan untuk bahan titrasi
Adapun cara membuat larutan NaOH 0,1 N dengan cara : menimbang 1 gr NaOH dan larutkan dengan aquades dalam beker glas (diaduk-aduk sampai homogeny). Larutan kemudian dimasukan ke dalam labu ukur 250ml, tambahkan aquades sampai batas dan dikocok sampai homogen.
Larutan NaOH 0,1 N tersebut dimasukan ke dalam buret 25 ml sampai titik nol.
Menimbang 0,315 gr asam oksalat (H2C2O4.H2O).
Asam oksalat dimasukkan ke dalam gelas beker, tambahkan aquades dan diaduk sampai homogen, pindahkan ke dalam labu ukur 50 ml dan tambahkan aquades sampai batas lalu di kocok supaya homogen.
Memipet sebanyak 10 ml larutan asam oksalat, masukan ke dalam Erlenmeyer, tambahkan indicator pp 3 tetes dan dititrasi dengan larutan NaOH sampai terjadi peerubahan warna.
Mengulangi langkah 7 sebanyak 3 kali
Mencatat informasi yang di dapat.
Penetapan kadar asam asetat dan cuka makan
Memipet sampel Asam asetat dan cuka makan sebanyak 10 ml
Diencerkan dengan penambahhan aquades,di dalam labu ukur 100ml sampai batas.
Kocok sampai homogen.
Memipet larutan sebanyak 10 ml dan tabahkan 3 tetes indicator pp di dalam Erlenmeyer.
Titrasi dengan NaOH sampai terjadi perubahan warna.
Mengulang langkah ke 4 sebanyak 3 kali
Mencatat volume NaOH dan informasi lain yang diperoleh.

HASIL ANALISIS
Data Hasi Analisis
Tabel standarisasi larutan NaOH
Keterangan:
Berat NaOH :1,0048 gram
Berat H2C204 . 2H2O :0,3155 gram


titrasi Volume asam oksalat (ml) Volume NaOH (ml) Perubahan warna
1 10 9,5 Tak berwarna-ungu muda
2 10 10,3 Tak berwarna-ungu muda

Tabel penetapan kadar asam asetat (CH3COOH)

titrasi Volume asam asetat (ml) Volume NaOH (ml) Perubahan warna
1 10 15,7 Tak berwarna-merah muda
2 10 14,6 Tak berwarna-merah muda

Table penetapan kadar asam cuka makan

titrasi Volume asam cuka (ml) Volume NaOH (ml) Perubahan warna
1 10 10,5 Tak berwarna-ungu tua
2 10 11,0 Tak berwarna-ungu tua




Rumus Penetapan Kadar

Normalitas NaOH hasil standarisasi :
N titran .V titran = N titrat .V titrat
N NaOH x V NaOH = N H2C2O4.2H2O x V H2C2O4.2H2O

N NaOH= (N H2C2O4.2H2O x V H2C2O4.2H2O)/(V NaOH)
Normalitas asam asetat
N titran .V titran = N titrat .V titrat
N CH3COOH x V CH3COOH = N NaOH x V NaOH
N CH3COOH=(N NaOH x V NaOH)/(V CH3COOH)

Kadar asam asetat :
Kadar CH_3 COOH (%b/v)= (N CH_3 COOH .V CH_3 COOH.BE CH_3 COOH)/(Volume sampel (L))


Perhitungan Kadar

Normalitas Asam oksalat
N = n x M
= gr/(BE x V (liter))
=0,315/(126/2 x 0,05)=(0,315 )/(63 x 0,05)=0,1 N
Normalitas NaOH

N NaOH= (N H2C2O4.2H2O x V H2C2O4.2H2O)/(V NaOH)
N NaOH=(0,1 x 10)/9,9=0,101 N


Normalitas asam asetat (CH3COOH)
Grek CH3COOH = grek NaOH
(N x V) CH3COOH = (N x V) NaOH
N CH3COOH=(N NaOH x V NaOH)/(V CH3COOH)
N CH3COOH = (0,101 x 15,15)/10=0,153 N

Normalitas Asam Cuka dengan bantuan Indikator
Grek Asam cuka = grek NaOH
(N x V) Asam cuka = (N x V) NaOH
N asam cuka=(N NaOH x V NaOH)/(V asam cuka)
=(0,101 x 10,75)/10=1,0857/10=0,1086 N

Kadar asam asetat

Kadar CH_3 COOH (%b/v)= (N CH_3 COOH .V CH_3 COOH.BE CH_3 COOH)/(Volume sampel (L) )

Kadar CH_3 COOH (%b/v)=(0,153 x 0,010 x 60)/0,010

Kadar CH_3 COOH (%b/v)=9,18 %

Kadar asam cuka

Kadar asam cuka = N CH_3 COOH. V CH_3 COOH . BE CH_3 COOH / V Sampel
=(N.asam cuka x V asam cuka X BE asam cuka)/(volume sampel)
=(0,1086 x 0,010 x 60)/0,010
=6,516 %




PEMBAHASAN

Dalam praktikum ini dipilih asam oksalat sebagai larutan standar karena asam oksalat memiliki berat ekuivalen (BE) yang besar (126) sehingga tidak mudah terpengaruh kemurniannya (asam oksalat tidak begitu higroskopis).
Larutan NaOH perlu distandarisasi terlebih dahulu untuk mengetahui normalitas NaOH yang sesungguhnya yang akan digunakan sebagai titran sehingga perhitungan yang didapat akan lebih akurat. Di samping itu, larutan NaOH bersifat higoskopis sehingga standarisasi menjadi proses yang harus di lakukan (dmi meminimalisir kesalahan analisis).
Titrasi dilakukan berulang-ulang (2 kali) untuk mendapatkan perbandingan hasil yang lebih akurat digunakan perhitungan rata-rata (lebih banyak dilakukan titrasi data yang dihasilkan akan semakin akurat).
Digunakan indicator Fenolphtalein karena Fenolphtalein tergolong asam yang sangat lemah, dalam keadaan yang tidak terionisasi indikator tersebut tidak berwarna. Jika dalam lingkungan basa fenolphtalein akan terionisasi lebih banyak dan memberikan warna terang karena anionnya (Day, 1981). Campuran karbonat dan hidroksida, atau karbonat dan bikarbonat, dapat ditetapkan dengan titrasi dengan menggunakan indikator fenolphtalein dan jingga metil (Day, 1981).
Standarisasi larutan NaOH
Langkah ini dilakukan untuk mengetahui normalitas larutan NaOH yang akan digunakan untuk analisa kuantitatif pada titrimetri yang akan dilakukan. Penjelasan langkah kerjanya sebagai berikut :
Membuat larutan NaOH 0,1 N yang mana di fungsikan untuk bahan titrasi
Adapun cara membuat larutan NaOH 0,1 N dengan cara : menimbang 1 gr NaOH dan larutkan dengan aquades dalam beker glas (diaduk-aduk sampai homogeny). Larutan kemudian dimasukan ke dalam labu ukur 250ml, tambahkan aquades sampai batas dan dikocok sampai homogen.
Larutan NaOH 0,1 N tersebut dimasukan ke dalam buret 25 ml sampai titik nol.
Menimbang 0,315 gr asam oksalat (H2C2O4.H2O) dengan menggunakan neraca analitik. Pada saat penimbangan neraca harus ditutup rapat karena bila terbuka akan terpengaruh oleh udara ataupun angin yang berhembus sehingga data yeng dihasilkan tidak sesuai. Neraca
Memasukan asam oksalat ke dalam gelas beker, aquades ditambahkan dan diaduk sampai homogen lalu dipindahkan ke dalam labu ukur 50 m, tambahkan aquades sampai batas kemudian larutan dikocok-kocok beberapa saat supaya benar-benar tercampur dengan aquades secara sempurna.
Asam oksalat dipipet 10 ml dengan pipet volume dan masukkan ke dalam Erlenmeyer. Tambahkan indicator pp 2-3 tetes dan titrasi segera dilakukan supaya larutan benar-benar belum mengalami perubahan.
Lakukan titrasi dengan tetesan pelan pada buret, karena setelah trjadi ekivalen penambahan sedikit titran akan menyebabkan perubahan pH yang besar. Titrasi dilakukan 2 kali, ambil volume NaOH rata-rata dan catat semua informasi yang diperoleh.
Data yang diperoleh bisa berbeda –beda karena:
Ketelitian dan keterampilan yang terbatas dan berbeda –beda dari masing-masing praktikan.
Praktikan selalu bergantian dalam melakukan percobaan.
Pembuatan larutan yang kurang baik
Kebersihan alat dan lain-lain

Penetapan kadar asam asetat dan cuka makan
Memipet sampel Asam asetat dan cuka makan sebanyak 10 ml dengan pipet gondok.
Diencerkan dengan penambahhan aquades di dalam labu ukur 100ml sampai batas.
Sesuai dengan judul pratikum yaitu alkalimetri bahasanya untuk penetapan kadar asam dibutuhkan standar senyawa basa.
Asam asetat merupakan asam lemah dan NaOH basa kuat jadi pada saat titik ekuvalen larutan menjadi netral (reaksi ini tergolog reaksi netralisasi).
Saat titik ekuivalen titrasi harus segare dihentikan karena penambahan sedikt saja NaOH akan menyebabkan perubahan pH yang besar.
Data yang diperoleh bisa berbeda-beda karena :
Ketelitian dan keterampilan yang terbatas dan berbeda –beda,
Praktikan selalu bergantian dalam percobaan
Pembuatan larutan yang kurang baik
Kebersihan alat dan lain-lain

Penjelasan Mekanisme Reaksi yang Terjadi
Reaksi antara asam asetat(CH3COOH) dengan NaOH
CH3COOH+ NaOH CH3COONa+H2O
Reaksi antara Asam oksalat dengan NaOH
H2C2O4.2H2O + 2NaOH Na2C2O4 + 4H2O

KESIMPULAN

Kesimpulan dari kegiatan praktikm alkalimetri ini adalah :
Alkalimetri merupakan metode penetapan kadar secara kuantitatif suatu senyawa yang bersifat asam dengan menggunakan senyawa basa standar.
Pemilihan indicator harus sesuai dengan titrasi yang dilakukan.
Penambahan sedikit titran pada saat titik ekivalen mengakibatkan perubahan pH secara drastis.
Pada titrasi asam basa n yang harus di hitung adalah jumlah H+ atau OH- yang dilepas atau diterima.
Dari hasil perhitungan, normalitas NaOH adalah 0,101 N, normalitas CH3COOH adalah 0,153 N dan normalitas asam cuka makan adalah 0,1086 N
Kadar CH3COOH adalah 9,18 % dan Kadar asam cuka makan adalah 6,516 %

DAFTAR PUSTAKA
Day,RA.,Uderwood A.L…1980.analisa kimia kuntitatif edisi keempat. erlangga: Jakarta.
Sya’bani,M.W.2009.Buku Petunjuk Pratikum Kimia Analisis. Akademi Teknoloi Kulit: Yogyakarta.
Titrasi.www.google.com diakses tanggal 13 maret 2010

;

No comments:

Post a Comment